neitheir $L^p((0, infty),mathbb{C}) subset L^q((0, infty),mathbb{C})$ nor $L^q((0, infty),mathbb{C}) subset...











up vote
0
down vote

favorite












I have to show that for $1le p < q <infty$

neitheir
$L^p((0, infty),mathbb{C}) subset L^q((0, infty),mathbb{C})$

nor
$L^q((0, infty),mathbb{C}) subset L^p((0, infty),mathbb{C})$.



I previously had to show, that there is no $a in mathbb{R}$ with which the function f:$(0, infty) to mathbb{R}$ f(x)=x^a

would be lebesgue integrable and there is a hint that i should use this information, but i have no idea how to do so.



I would be very thankfull for any kind of help.










share|cite|improve this question


























    up vote
    0
    down vote

    favorite












    I have to show that for $1le p < q <infty$

    neitheir
    $L^p((0, infty),mathbb{C}) subset L^q((0, infty),mathbb{C})$

    nor
    $L^q((0, infty),mathbb{C}) subset L^p((0, infty),mathbb{C})$.



    I previously had to show, that there is no $a in mathbb{R}$ with which the function f:$(0, infty) to mathbb{R}$ f(x)=x^a

    would be lebesgue integrable and there is a hint that i should use this information, but i have no idea how to do so.



    I would be very thankfull for any kind of help.










    share|cite|improve this question
























      up vote
      0
      down vote

      favorite









      up vote
      0
      down vote

      favorite











      I have to show that for $1le p < q <infty$

      neitheir
      $L^p((0, infty),mathbb{C}) subset L^q((0, infty),mathbb{C})$

      nor
      $L^q((0, infty),mathbb{C}) subset L^p((0, infty),mathbb{C})$.



      I previously had to show, that there is no $a in mathbb{R}$ with which the function f:$(0, infty) to mathbb{R}$ f(x)=x^a

      would be lebesgue integrable and there is a hint that i should use this information, but i have no idea how to do so.



      I would be very thankfull for any kind of help.










      share|cite|improve this question













      I have to show that for $1le p < q <infty$

      neitheir
      $L^p((0, infty),mathbb{C}) subset L^q((0, infty),mathbb{C})$

      nor
      $L^q((0, infty),mathbb{C}) subset L^p((0, infty),mathbb{C})$.



      I previously had to show, that there is no $a in mathbb{R}$ with which the function f:$(0, infty) to mathbb{R}$ f(x)=x^a

      would be lebesgue integrable and there is a hint that i should use this information, but i have no idea how to do so.



      I would be very thankfull for any kind of help.







      lp-spaces






      share|cite|improve this question













      share|cite|improve this question











      share|cite|improve this question




      share|cite|improve this question










      asked Nov 18 at 10:25









      Zweistein

      11




      11






















          1 Answer
          1






          active

          oldest

          votes

















          up vote
          0
          down vote













          Try this: if $p<q$ pick $p<r<q$ and consider the function $f:(0,+infty)tomathbb{R}$ given by
          $$
          f(x)=x^{-frac{1}{r}}chi_{(0,1)}(x)
          $$

          it's easy to show that $fin L^p(0,+infty)$ but $fnotin L^q(0,+infty)$. For the second part you might just take
          $$
          g(x)=x^{-frac{1}{r}}chi_{(1,+infty)}(x)
          $$

          Here $fin L^q(0,+infty)$ but $fnotin L^p(0,+infty)$. This example should show you that in general for $p<q$ you have $L^qsubset L^p$ if the space you are considering have "measure mass condensed in bounded sets" (not formal, but in a suitable sense it works), for example in $[0,1]$. If the mass is "condensed at $infty$", for example with $mathbb{N}$ and the counting measure, then is reversed: $ell^psubsetell^q$.






          share|cite|improve this answer























            Your Answer





            StackExchange.ifUsing("editor", function () {
            return StackExchange.using("mathjaxEditing", function () {
            StackExchange.MarkdownEditor.creationCallbacks.add(function (editor, postfix) {
            StackExchange.mathjaxEditing.prepareWmdForMathJax(editor, postfix, [["$", "$"], ["\\(","\\)"]]);
            });
            });
            }, "mathjax-editing");

            StackExchange.ready(function() {
            var channelOptions = {
            tags: "".split(" "),
            id: "69"
            };
            initTagRenderer("".split(" "), "".split(" "), channelOptions);

            StackExchange.using("externalEditor", function() {
            // Have to fire editor after snippets, if snippets enabled
            if (StackExchange.settings.snippets.snippetsEnabled) {
            StackExchange.using("snippets", function() {
            createEditor();
            });
            }
            else {
            createEditor();
            }
            });

            function createEditor() {
            StackExchange.prepareEditor({
            heartbeatType: 'answer',
            convertImagesToLinks: true,
            noModals: true,
            showLowRepImageUploadWarning: true,
            reputationToPostImages: 10,
            bindNavPrevention: true,
            postfix: "",
            imageUploader: {
            brandingHtml: "Powered by u003ca class="icon-imgur-white" href="https://imgur.com/"u003eu003c/au003e",
            contentPolicyHtml: "User contributions licensed under u003ca href="https://creativecommons.org/licenses/by-sa/3.0/"u003ecc by-sa 3.0 with attribution requiredu003c/au003e u003ca href="https://stackoverflow.com/legal/content-policy"u003e(content policy)u003c/au003e",
            allowUrls: true
            },
            noCode: true, onDemand: true,
            discardSelector: ".discard-answer"
            ,immediatelyShowMarkdownHelp:true
            });


            }
            });














             

            draft saved


            draft discarded


















            StackExchange.ready(
            function () {
            StackExchange.openid.initPostLogin('.new-post-login', 'https%3a%2f%2fmath.stackexchange.com%2fquestions%2f3003355%2fneitheir-lp0-infty-mathbbc-subset-lq0-infty-mathbbc-nor%23new-answer', 'question_page');
            }
            );

            Post as a guest















            Required, but never shown

























            1 Answer
            1






            active

            oldest

            votes








            1 Answer
            1






            active

            oldest

            votes









            active

            oldest

            votes






            active

            oldest

            votes








            up vote
            0
            down vote













            Try this: if $p<q$ pick $p<r<q$ and consider the function $f:(0,+infty)tomathbb{R}$ given by
            $$
            f(x)=x^{-frac{1}{r}}chi_{(0,1)}(x)
            $$

            it's easy to show that $fin L^p(0,+infty)$ but $fnotin L^q(0,+infty)$. For the second part you might just take
            $$
            g(x)=x^{-frac{1}{r}}chi_{(1,+infty)}(x)
            $$

            Here $fin L^q(0,+infty)$ but $fnotin L^p(0,+infty)$. This example should show you that in general for $p<q$ you have $L^qsubset L^p$ if the space you are considering have "measure mass condensed in bounded sets" (not formal, but in a suitable sense it works), for example in $[0,1]$. If the mass is "condensed at $infty$", for example with $mathbb{N}$ and the counting measure, then is reversed: $ell^psubsetell^q$.






            share|cite|improve this answer



























              up vote
              0
              down vote













              Try this: if $p<q$ pick $p<r<q$ and consider the function $f:(0,+infty)tomathbb{R}$ given by
              $$
              f(x)=x^{-frac{1}{r}}chi_{(0,1)}(x)
              $$

              it's easy to show that $fin L^p(0,+infty)$ but $fnotin L^q(0,+infty)$. For the second part you might just take
              $$
              g(x)=x^{-frac{1}{r}}chi_{(1,+infty)}(x)
              $$

              Here $fin L^q(0,+infty)$ but $fnotin L^p(0,+infty)$. This example should show you that in general for $p<q$ you have $L^qsubset L^p$ if the space you are considering have "measure mass condensed in bounded sets" (not formal, but in a suitable sense it works), for example in $[0,1]$. If the mass is "condensed at $infty$", for example with $mathbb{N}$ and the counting measure, then is reversed: $ell^psubsetell^q$.






              share|cite|improve this answer

























                up vote
                0
                down vote










                up vote
                0
                down vote









                Try this: if $p<q$ pick $p<r<q$ and consider the function $f:(0,+infty)tomathbb{R}$ given by
                $$
                f(x)=x^{-frac{1}{r}}chi_{(0,1)}(x)
                $$

                it's easy to show that $fin L^p(0,+infty)$ but $fnotin L^q(0,+infty)$. For the second part you might just take
                $$
                g(x)=x^{-frac{1}{r}}chi_{(1,+infty)}(x)
                $$

                Here $fin L^q(0,+infty)$ but $fnotin L^p(0,+infty)$. This example should show you that in general for $p<q$ you have $L^qsubset L^p$ if the space you are considering have "measure mass condensed in bounded sets" (not formal, but in a suitable sense it works), for example in $[0,1]$. If the mass is "condensed at $infty$", for example with $mathbb{N}$ and the counting measure, then is reversed: $ell^psubsetell^q$.






                share|cite|improve this answer














                Try this: if $p<q$ pick $p<r<q$ and consider the function $f:(0,+infty)tomathbb{R}$ given by
                $$
                f(x)=x^{-frac{1}{r}}chi_{(0,1)}(x)
                $$

                it's easy to show that $fin L^p(0,+infty)$ but $fnotin L^q(0,+infty)$. For the second part you might just take
                $$
                g(x)=x^{-frac{1}{r}}chi_{(1,+infty)}(x)
                $$

                Here $fin L^q(0,+infty)$ but $fnotin L^p(0,+infty)$. This example should show you that in general for $p<q$ you have $L^qsubset L^p$ if the space you are considering have "measure mass condensed in bounded sets" (not formal, but in a suitable sense it works), for example in $[0,1]$. If the mass is "condensed at $infty$", for example with $mathbb{N}$ and the counting measure, then is reversed: $ell^psubsetell^q$.







                share|cite|improve this answer














                share|cite|improve this answer



                share|cite|improve this answer








                edited Nov 18 at 10:56

























                answered Nov 18 at 10:50









                Marco

                1909




                1909






























                     

                    draft saved


                    draft discarded



















































                     


                    draft saved


                    draft discarded














                    StackExchange.ready(
                    function () {
                    StackExchange.openid.initPostLogin('.new-post-login', 'https%3a%2f%2fmath.stackexchange.com%2fquestions%2f3003355%2fneitheir-lp0-infty-mathbbc-subset-lq0-infty-mathbbc-nor%23new-answer', 'question_page');
                    }
                    );

                    Post as a guest















                    Required, but never shown





















































                    Required, but never shown














                    Required, but never shown












                    Required, but never shown







                    Required, but never shown

































                    Required, but never shown














                    Required, but never shown












                    Required, but never shown







                    Required, but never shown







                    Popular posts from this blog

                    Le Mesnil-Réaume

                    Ida-Boy-Ed-Garten

                    web3.py web3.isConnected() returns false always